Szósty element: rozwiązanie zagadki

https://xpil.eu/ymn

3, 10, 93, 196, 485 - i co dalej? Jaka zależność łączy te liczby?

Okazało się, że rozwiązań jest kilka, na co jednak z właściwą sobie beztroską nie wpadłem przy pisaniu zagadki. Wiadomo przecież, że dla dowolnych n liczb da się napisać wielomian (n-1)-go stopnia, który będzie miał pierwiastki równe tym liczbom. NIektórzy z Was uświadomili mi to, przysyłając takie wielomiany. Touché!

Na pomysł, aby użyć tych liczb jako wykładników jako pierwszy wpadł Waldek, któremu w związku z tym należy się palma zwycięzcy oraz dożywotnie prawo przechwalania się rozwiązaniem zagadki 🙂

Zobaczmy co dzieje się jeżeli zaczniemy kolejno podnosić dwójkę do potęg podanych w zagadce:

\(2^{3}=8\\2^{10}=1 024\\2^{93}=9 903 520 314 283 042 199 192 993 792\\2^{196}=10 043 362 776 618 (...) 206 336\\2^{485}=99 895 953 610 111 (...) 285 632\)

Coś się zaczyna klarować...

Okazuje się, że każda z tych potęg dwójki jest odrobinę bliższa (procentowo) całkowitej potędze liczby dziesięć od swojej poprzedniczki. Między \(2^3\) a 10 jest 20% różnicy. Między \(2^{10}\) a 1000 - już tylko 2.4%, między \(2^{93}\) a \(10^{28}\) tylko niecały 1% i tak dalej. Pomiędzy \(2^{10}\) a \(2^{93}\) nie ma ani jednej potęgi dwójki, która byłaby bliższa potędze 10 o mniej niż 2.4%.

Między \(2^{485}\) a \(10^{146}\) jest około 0.1% różnicy.

Kolejną taką liczbą jest więc - oczywiście - 2136.

2136

Różnica między \(2^{2136}\) a \(10^{643}\) to zaledwie 0.016%.

A co dalej?

Jak wspomniałem w treści zagadki, zawsze da się znaleźć potęgę dwójki, która będzie procentowo bliższa jakiejś całkowitej potędze 10 niż jej poprzedniczka. Jest to więc ciąg nieskończony. Kilka kolejnych wyrazów: 13301, 28738, 42039, 70777, 254370, 325147, 6107016, 6432163, 44699994, 51132157, 146964308, 198096465, 345060773, 1578339557, 1923400330, 82361153417, 496090320832, 578451474249, 2809896217828, 6198243909905.

Więcej szczegółów można znaleźć tutaj.

https://xpil.eu/ymn

2 komentarze

  1. Sprytne. Czyli mówimy tutaj o funckji f(x, y) = (10^x – 2^y) / 10^x, gdzie Twój ciąg składa się z wartości y dających lokalne minima tej funkcji…?

    A nie, uprościłem. Bo Ty nie jesteś zainteresowany lokalnymi minimami, ale wymagasz, żeby każde kolejne minimum było mniejsze od poprzedniego. Czyli mówimy o jakimś podzbiorze wartości y z poprzedniego akapitu?

    Przepraszam, jeżeli pomieszałem, ale problem wydaje mi się na tyle ciekawy, żeby coś o nim napisać, ale nie na tyle pilny, żeby analizować go tu i teraz na papierze. 🙂

    1. Tak naprawdę da się to sprowadzić do szukania kolejnych przybliżeń log(2,10) w postaci ułamków o całkowitych licznikach i mianownikach. Log(2,10) wynosi mniej więcej 3.321928094887362347870319429489390175864831393024580612054, a kolejne ułamki dające coraz lepsze przybliżenie tej wartości to 3/1, 10/3, 93/28, 196/59, 485/146, 2136/643 i tak dalej. Jak widać ciąg w tej zagadce to ciąg liczników tych ułamków. Mianowniki z kolei odpowiadają wykładnikom dziesiątki.

Leave a Comment

Komentarze mile widziane.

Jeżeli chcesz do komentarza wstawić kod, użyj składni:
[code]
tutaj wstaw swój kod
[/code]

Jeżeli zrobisz literówkę lub zmienisz zdanie, możesz edytować komentarz po jego zatwierdzeniu.